Welcome to the MacNN Forums.

If this is your first visit, be sure to check out the FAQ by clicking the link above. You may have to register before you can post: click the register link above to proceed. To start viewing messages, select the forum that you want to visit from the selection below.

You are here: MacNN Forums > Community > MacNN Lounge > Probability Question

Probability Question
Thread Tools
strictlyplaid
Senior User
Join Date: Jun 2004
Status: Offline
Reply With Quote
Nov 1, 2005, 10:17 PM
 
How can I be the first post in a thread I didn't start?
     
realmeatychunks
Junior Member
Join Date: Jun 2003
Status: Offline
Reply With Quote
Nov 1, 2005, 10:29 PM
 
Why isn't my Post there? Strange.

Okay, well here is what I tried to post:


5.12 from Wackerly et al.

Suppose that the random variables X and Y have joint probability density function f(x,y) given by

f(x,y) = 6(x^2)y for 0≤x≤y, x+y≤2
0 elsewhere.

a) Verify that this is a valid joint density function.

This means that f(x,y) is greater than 0 for all x,y which is trivial. The second part, is that the double integral of f(x,y) over all space is equal to 1. I'm having trouble setting the limits for the integration, at first I tried setting the limits for the x integration from 0 to y and for y from 0 to 2, which yielded the wrong result (64/5). I've tried several others and cannot get the integral equal 1. Does anyone have any insight as to what the limits of integration should be and why?

Thanks in advance,
Greg

(edited to fix the function formating [thanks hyperb0le] and to make my post a little clearer)
( Last edited by realmeatychunks; Nov 2, 2005 at 12:56 AM. )
     
11011001
Mac Elite
Join Date: May 2001
Location: Up north
Status: Offline
Reply With Quote
Nov 1, 2005, 11:24 PM
 
edit: neat
     
hyperb0le
Mac Enthusiast
Join Date: Mar 2003
Location: Atlanta, GA
Status: Offline
Reply With Quote
Nov 1, 2005, 11:36 PM
 
Is the function supposed to be f(x,y) = 6x^(2y) or f(x,y) = 6x^2 * y?

EDIT: Judging by your result (64/5), I assume that it is f(x,y) = 6x^2 * y. My initial instinct is to set the limits on the y integration from 0 to (2-x) and set the limits for x from 0 to 2 (x and y axis are bounds for the region, as is y=2-x (or x+y=2), and so is the vertical line x=2 where y=2-x intersects the x-axis) and evaluate the y integral first. However, that results in 16/5... I may be missing something, as I am very tired, but those seem like the correct limits. Is there any possiblity that the function is not a valid joint density function?
( Last edited by hyperb0le; Nov 1, 2005 at 11:57 PM. )
     
realmeatychunks
Junior Member
Join Date: Jun 2003
Status: Offline
Reply With Quote
Nov 2, 2005, 12:25 AM
 
Ah yes, sorry about the 'format' of the function, you are correct, it is 6(x^2)y. I had written it as this, but did not correct it when i reposted it. I suppose it is possible that it is not a valid distribution function, but the next part of the question asks for the probability that X + Y < 1.

This makes me think it is more probable (haha) that it is indeed a valid distribution function and there's just something I'm not getting.
     
strictlyplaid  (op)
Senior User
Join Date: Jun 2004
Status: Offline
Reply With Quote
Nov 2, 2005, 01:35 AM
 
Originally Posted by realmeatychunks
Why isn't my Post there? Strange.

Okay, well here is what I tried to post:


5.12 from Wackerly et al.

Suppose that the random variables X and Y have joint probability density function f(x,y) given by

f(x,y) = 6(x^2)y for 0≤x≤y, x+y≤2
0 elsewhere.

a) Verify that this is a valid joint density function.

This means that f(x,y) is greater than 0 for all x,y which is trivial. The second part, is that the double integral of f(x,y) over all space is equal to 1. I'm having trouble setting the limits for the integration, at first I tried setting the limits for the x integration from 0 to y and for y from 0 to 2, which yielded the wrong result (64/5). I've tried several others and cannot get the integral equal 1. Does anyone have any insight as to what the limits of integration should be and why?

Thanks in advance,
Greg

(edited to fix the function formating [thanks hyperb0le] and to make my post a little clearer)
My double integral skills are a little weak, but if I'm picturing this correctly, you're integrating over a triangle-shaped area in Real(2) space. You set the bounds of y-integration from x to 2-x, and the bounds of x integration from 0 to 1 (the point at which these two lines intersect). Essentially, you're adding up a series of slices of a triangle over the desired area; plotting this out makes it clear what you're doing. Maple tells me that this integral = 1, the desired answer.

Hope that helps!
     
ghporter
Administrator
Join Date: Apr 2001
Location: San Antonio TX USA
Status: Offline
Reply With Quote
Nov 2, 2005, 09:54 AM
 
Originally Posted by strictlyplaid
How can I be the first post in a thread I didn't start?
Stupid database tricks, of course! I'm sorry to say that while the version upgrade has been sorted out, the server is still an issue.

I was going to make this a probability joke (quantum states and all that) but it just seemed too off topic.

Glenn -----OTR/L, MOT, Tx
     
realmeatychunks
Junior Member
Join Date: Jun 2003
Status: Offline
Reply With Quote
Nov 2, 2005, 06:23 PM
 
Thank you for the explanation strictlyplaid. I think you are correct.

Your help is much appreciated.

Greg
     
tie
Professional Poster
Join Date: Feb 2001
Status: Offline
Reply With Quote
Nov 2, 2005, 08:21 PM
 
Keep the thread alive! What if we have a distribution on AxB such that p(a,b) = Theta(p(a)p(b)). Do there exist distributions q_s and q_s' (parametrized by s) and t(s), such that p(a,b) = integral t(s) q_s(a) q_s'(b) such that q_s(a) = Theta(p(a)), q_s'(b) = Theta(p(b)) and of course integral t(s) = 1?
     
strictlyplaid  (op)
Senior User
Join Date: Jun 2004
Status: Offline
Reply With Quote
Nov 3, 2005, 12:26 AM
 
Originally Posted by realmeatychunks
Thank you for the explanation strictlyplaid. I think you are correct.

Your help is much appreciated.

Greg
No problem!
     
11011001
Mac Elite
Join Date: May 2001
Location: Up north
Status: Offline
Reply With Quote
Nov 3, 2005, 01:03 AM
 
Originally Posted by realmeatychunks
Why isn't my Post there? Strange.
5.12 from Wackerly et al.
I'm going to guess that you might be from Calgary?
     
realmeatychunks
Junior Member
Join Date: Jun 2003
Status: Offline
Reply With Quote
Nov 3, 2005, 02:00 AM
 
Nope, I'm not from Calgary, what gave you that impression?

I go to university in Montréal, but am originally from New Hampshire.
     
11011001
Mac Elite
Join Date: May 2001
Location: Up north
Status: Offline
Reply With Quote
Nov 3, 2005, 04:27 AM
 
Originally Posted by realmeatychunks
Nope, I'm not from Calgary, what gave you that impression?

I go to university in Montréal, but am originally from New Hampshire.
I have the same textbook, though I realize that many different universities use it, there was that slim chance. At least you are in Canada. Booya!
     
chabig
Addicted to MacNN
Join Date: Jun 1999
Location: Las Vegas, NV, USA
Status: Offline
Reply With Quote
Nov 3, 2005, 09:50 AM
 
Originally Posted by strictlyplaid
My double integral skills are a little weak, but if I'm picturing this correctly, you're integrating over a triangle-shaped area in Real(2) space. You set the bounds of y-integration from x to 2-x, and the bounds of x integration from 0 to 1 (the point at which these two lines intersect). Essentially, you're adding up a series of slices of a triangle over the desired area; plotting this out makes it clear what you're doing. Maple tells me that this integral = 1, the desired answer.

Hope that helps!
Aren't we talking about an equilateral triangle with vertices at (0,0), (1,1), and (2,0)? If so, then I would calculate 2 integrals; one covering 0≤x≤1 (integrate y from 0 to x), and the other covering 1≤x≤2 (integrate y from 0 to 2-x).

Chris
     
strictlyplaid  (op)
Senior User
Join Date: Jun 2004
Status: Offline
Reply With Quote
Nov 3, 2005, 10:28 AM
 
Originally Posted by chabig
Aren't we talking about an equilateral triangle with vertices at (0,0), (1,1), and (2,0)? If so, then I would calculate 2 integrals; one covering 0≤x≤1 (integrate y from 0 to x), and the other covering 1≤x≤2 (integrate y from 0 to 2-x).

Chris
Actually, the triangle has vertices (0,0), (0,2), and (1,1). Your integral is also improper (it integrates to more than one.)

Incidentally, a more elegant solution to the integration problem that you are defining would be to integrate x from y to 2-y, then integrate y from 0 to 1.
     
iREZ
Professional Poster
Join Date: Dec 2003
Location: Los Angeles of the East
Status: Offline
Reply With Quote
Nov 3, 2005, 01:55 PM
 
what a bunch of nerds!!! (fixes jock strap)
NOW YOU SEE ME! 2.4 MBP and 2.0 MBP (running ubuntu)
     
chabig
Addicted to MacNN
Join Date: Jun 1999
Location: Las Vegas, NV, USA
Status: Offline
Reply With Quote
Nov 3, 2005, 09:00 PM
 
Originally Posted by strictlyplaid
Actually, the triangle has vertices (0,0), (0,2), and (1,1). Your integral is also improper (it integrates to more than one.)

Incidentally, a more elegant solution to the integration problem that you are defining would be to integrate x from y to 2-y, then integrate y from 0 to 1.
I think I disagree. Your triangle would satisfy the inequalities x≥0, x≥y, and x+y≤2. But the problem given specified x≥0, x≤y, and x+y≤2. And it could be solved with by integrating x from y=x to y=2-y, while y ranges from 0 to 1.

Still, my integrals evaluate to 2.2, which either means I made a mistake with the integration, or the given function is not a valid joint density function. Since I've done the integration twice using different methods and I get the same result, I tend to think the latter is true.

Chris
( Last edited by chabig; Nov 3, 2005 at 09:10 PM. )
     
strictlyplaid  (op)
Senior User
Join Date: Jun 2004
Status: Offline
Reply With Quote
Nov 3, 2005, 09:30 PM
 
Originally Posted by chabig
I think I disagree. Your triangle would satisfy the inequalities x≥0, x≥y, and x+y≤2. But the problem given specified x≥0, x≤y, and x+y≤2. And it could be solved with by integrating x from y=x to y=2-y, while y ranges from 0 to 1.

Still, my integrals evaluate to 2.2, which either means I made a mistake with the integration, or the given function is not a valid joint density function. Since I've done the integration twice using different methods and I get the same result, I tend to think the latter is true.

Chris
This part of your reply is wrong: "Your triangle would satisfy the inequalities x≥0, x≥y, and x+y≤2. But the problem given specified x≥0, x≤y, and x+y≤2." Plot the lines y=x and y=2-x out on the Cartesian plane. shade everything above y=x (i.e., y>x) and everything below y=2-x (i.e., x+y<2 or otherwise written y<2-x). The overlap is the triangle described by vertices (0,0), (0,2), (1,1). This is the region that satisfies the conditions of the problem. The fact that my area integrates to 1 is further evidence; note that the problem asks you to prove THAT the density is proper, not determine WHETHER it is.
     
chabig
Addicted to MacNN
Join Date: Jun 1999
Location: Las Vegas, NV, USA
Status: Offline
Reply With Quote
Nov 3, 2005, 09:59 PM
 
Originally Posted by strictlyplaid
This part of your reply is wrong: "Your triangle would satisfy the inequalities x≥0, x≥y, and x+y≤2. But the problem given specified x≥0, x≤y, and x+y≤2." Plot the lines y=x and y=2-x out on the Cartesian plane. shade everything above y=x (i.e., y>x) and everything below y=2-x (i.e., x+y<2 or otherwise written y<2-x). The overlap is the triangle described by vertices (0,0), (0,2), (1,1). This is the region that satisfies the conditions of the problem. The fact that my area integrates to 1 is further evidence; note that the problem asks you to prove THAT the density is proper, not determine WHETHER it is.
I agree with your method of plotting y=x and y=2-x. I was doing the same thing. But I misread the original problem and shaded the area below y=x, not the area above it. And I do agree that the integral now equals 1.

Chris
     
strictlyplaid  (op)
Senior User
Join Date: Jun 2004
Status: Offline
Reply With Quote
Nov 3, 2005, 10:12 PM
 
Originally Posted by chabig
I agree with your method of plotting y=x and y=2-x. I was doing the same thing. But I misread the original problem and shaded the area below y=x, not the area above it. And I do agree that the integral now equals 1.

Chris
No prob, getting the inequalities right was the only really tricky part of the problem and it took me a try or two as well. :-)
     
chabig
Addicted to MacNN
Join Date: Jun 1999
Location: Las Vegas, NV, USA
Status: Offline
Reply With Quote
Nov 3, 2005, 10:31 PM
 
So the only question remaining is, did Greg get it?



Chris
     
realmeatychunks
Junior Member
Join Date: Jun 2003
Status: Offline
Reply With Quote
Nov 4, 2005, 12:05 AM
 
Originally Posted by chabig
So the only question remaining is, did Greg get it?



Chris
Nice image, LaTeX?

I do understand it, after reading the explanation and drawing out the integration region. Thanks a lot guys!

One further question though, if I were to take the marginal distribution function of y, defined as

fy = int(f(x,y),x,-inf,inf) i.e. integrate x over all space and hold y constant,

then would the integral int(f(x,y),x,0,y) be correct? (I don't think it is...) Oi, it's been awhile since i've done something like this. Maybe I need to perform a transformation of variables or something.

I know that the marginal distribution function of x = fx = int(f(x,y),y,x,2-x) which makes sense given our previous work (and it is confirmed by being a beta distribution as the book asks you to prove).

Regards,
Greg
     
chabig
Addicted to MacNN
Join Date: Jun 1999
Location: Las Vegas, NV, USA
Status: Offline
Reply With Quote
Nov 4, 2005, 01:13 AM
 
Originally Posted by realmeatychunks
Nice image, LaTeX?
No. Grapher (ships with OS X Tiger). I just wrote the formula and then took a screen capture using cmd-shift-4.

One further question though, if I were to take the marginal distribution function of y, defined as

fy = int(f(x,y),x,-inf,inf) i.e. integrate x over all space and hold y constant,

then would the integral int(f(x,y),x,0,y) be correct? (I don't think it is...) Oi, it's been awhile since i've done something like this. Maybe I need to perform a transformation of variables or something.

I know that the marginal distribution function of x = fx = int(f(x,y),y,x,2-x) which makes sense given our previous work (and it is confirmed by being a beta distribution as the book asks you to prove).
I think you're right. If the general equation is:



Then the specific equation would be:



I think you have to split the integral into two regions, though, because you have to define the limits of integration differently depending upon whether y is less than or greater than 1.

Chris
( Last edited by chabig; Nov 4, 2005 at 10:23 AM. )
     
bstone
Mac Elite
Join Date: Jun 2000
Location: Boston, MA
Status: Offline
Reply With Quote
Nov 6, 2005, 01:32 AM
 
Emergency Medicine & Urgent Care.
     
jcadam
Mac Elite
Join Date: Oct 2001
Location: Colorado Springs
Status: Offline
Reply With Quote
Nov 6, 2005, 04:24 AM
 
I took Calc I, II, & III, as well as linear algebra, discrete math/set theory, and adv. probability in College. I have not used them in the 3 years since I graduated. And I have consumed enough alcohol since then that the neurons that were storing that knowledge are most likely dead.
Caffeinated Rhino Software -- Education and Training management software
     
realmeatychunks
Junior Member
Join Date: Jun 2003
Status: Offline
Reply With Quote
Nov 7, 2005, 02:20 AM
 
Originally Posted by jcadam
I took Calc I, II, & III, as well as linear algebra, discrete math/set theory, and adv. probability in College. I have not used them in the 3 years since I graduated. And I have consumed enough alcohol since then that the neurons that were storing that knowledge are most likely dead.
What field are you in?
     
jcadam
Mac Elite
Join Date: Oct 2001
Location: Colorado Springs
Status: Offline
Reply With Quote
Nov 7, 2005, 02:38 PM
 
Oh, there you go getting me started <RANT>

I majored in Computer Science (minor in Math).

Used an Army scholarship to fund my education, so now I'm Uncle Sam's indentured slave until May 2006 (almost there w00t). I'm a signal corps officer (communications), which means I could be doing something IT/tech related right now, but currently the Army is making maximum use of my BS in Comp Sci. by having me do Human Resources management (I'm an S-1) type work.

I'm looking forward to getting out so I can be a software engineer, if my unused 4 year old degree isn't totally worthless by now.

</RANT>
Caffeinated Rhino Software -- Education and Training management software
     
   
 
Forum Links
Forum Rules
You may not post new threads
You may not post replies
You may not post attachments
You may not edit your posts
BB code is On
Smilies are On
[IMG] code is On
HTML code is Off
Top
Privacy Policy
All times are GMT -4. The time now is 03:47 AM.
All contents of these forums © 1995-2017 MacNN. All rights reserved.
Branding + Design: www.gesamtbild.com
vBulletin v.3.8.8 © 2000-2017, Jelsoft Enterprises Ltd.,